Jump to content

sandokhan

Senior Members
  • Posts

    60
  • Joined

  • Last visited

Recent Profile Visitors

The recent visitors block is disabled and is not being shown to other users.

sandokhan's Achievements

Meson

Meson (3/13)

-8

Reputation

  1. It cannot be any poorer than Michelson's, who derived the WRONG equation (Coriolis effect) while claiming all the while it was the Sagnac effect formula. It cannot be any poorer than Einstein's, who made this statement in 1905: "The principle of the constancy of the velocity of light is of course contained in Maxwell's equations” However, those are the HEAVISIDE-LORENTZ equations, NOT the original J.C. MAXWELL equations which are invariant under galilean transformations. Dr. A.G. Kelly discovered the humongous fudging of the data in the Hafele-Keating experiment: http://www.cartesio-episteme.net/h%26kpaper.htm His credibility is as good as any of the top researchers in the field. Certainly he will include the CORIOLIS EFFECT formula as a substitution for the SAGNAC EFFECT formula, since this subject was not thoroughly investigated/researched at all in the 20th century. However, now there is no reason to do so anymore: we have a generalized/global SAGNAC EFFECT formula which essentially and beautifully captures the entire phenomenon. E.J. Post made the same comments as did Dr. Kelly. Sagnac Effect, E.J. Post, Reviews of Modern Physics, April 1967 "The search for a physically meaningful transformation is not aided in any way whatever by the principle of general space-time covariance, nor is it true that the space-time theory of gravitation plays any direct role in establishing physically correct transformations." Post (1967) shows that the two (Sagnac and STR) are of very different orders of magnitude. He says that the dilation factor to be applied under SR is “indistinguishable with presently available equipment” and “is still one order smaller than the Doppler correction, which occurs when observing fringe shifts” in the Sagnac tests. He also points out that the Doppler effect “is v/c times smaller than the effect one wants to observe." Here Post states that the effect forecast by SR, for the time dilation aboard a moving object, is far smaller than the effect to be observed in a Sagnac test. It is essential to understand that the SAGNAC EFFECT requires the use of the original MAXWELL equations to be properly described since the SAGNAC interferometer represents A TOPOLOGICAL OBSTRUCTION, hence requiring a higher topology. https://books.google.ro/books?id=qsOBhKVM1qYC&pg=PA85&dq=Lakhtakia+Barrett,+T.W.+electromagnetism&hl=ro&sa=X&ved=0ahUKEwiV3IXAmbnZAhXKJVAKHeebCnUQ6AEIJjAA#v=onepage&q=Lakhtakia%20Barrett%2C%20T.W.%20electromagnetism&f=falseDr. T.W. Barrett, "Electromagnetic Phenomena Not Explained by Maxwell's Equations" pg 6 - 85 https://www.researchgate.net/publication/288491190_SAGNAC_EFFECT_A_consequence_of_conservation_of_action_due_to_gauge_field_global_conformal_invariance_in_a_multiply-joined_topology_of_coherent_fields Dr. Terence W. Barrett (Stanford Univ., Princeton Univ., U.S. Naval Research Laboratory, Univ. of Edinburgh, author of over 200 papers on advanced electromagnetism):Topology has been used to provide answers to questions concerning what is most fundamental in physical explanation. That question itself implies the question concerning what mathematical structures one uses with confidence to adequately “paint” or describe physical models built from empirical facts. For example, differential equations of motion cannot be fundamental, because they are dependent on boundary conditions which must be justified—usually by group theoretical considerations. Perhaps, then, group theory is fundamental.Group theory certainly offers an austere shorthand for fundamental transformation rules. But it appears to the present writer that the final judge of whether a mathematical group structure can, or cannot, be applied to a physical situation is the topology of that physical situation. Topology dictates and justifies the group transformations. So for the present writer, the answer to the question of what is the most fundamental physical description is that it is a description of the topology of the situation. With the topology known, the group theory description is justified and equations of motion can then be justified and defined in specific differential equation form. If there is a requirement for an understanding more basic than the topology of the situation, then all that is left is verbal description of visual images. So we commence an examination of electromagnetism under the assumption that topology defines group transformations and the group transformation rules justify the algebra underlying the differential equations of motion.Those situations in which the Aμ potentials are measurable possess a topology, the transformation rules of which are describable by the SU(2) group; and those situations in which the Aμ potentials are not measurable possess a topology, the transformation rules of which are describable by the U(1) group.https://pdfs.semanticscholar.org/a9bc/aee223173c4fef38a36623c550a05c584801.pdfTopology and the Physical Properties of Electromagnetic Fields
  2. That article is available elsewhere for full reading. It is also discussed in several other works on the subject. It is fundamental in understanding the subject of the first part of your message. Here is Carl Ockert's analysis of the Fizeau experiment: http://adsabs.harvard.edu/abs/1968AmJPh..36..158O http://adsabs.harvard.edu/abs/1969AmJPh..37..335O
  3. You haven't done your homework on this one. Sagnac Effect, E.J. Post, Reviews of Modern Physics, April 1967 "The search for a physically meaningfull transformation is not aided in any way whatever by the principle of general space-time covariance, nor is it true that the space-time theory of gravitation plays any direct role in establishing physically correct transformations." The Sagnac effect is a non-relativistic effect. COMPARISON OF THE SAGNAC EFFECT WITH SPECIAL RELATIVITY, starts on page 7, calculations/formulas on page 8http://www.naturalphilosophy.org/pdf/ebooks/Kelly-TimeandtheSpeedofLight.pdfpage 8Because many investigators claim that theSagnac effect is made explicable by using theTheory of Special Relativity, a comparison ofthat theory with the actual test results is givenbelow. It will be shown that the effectscalculated under these two theories are of verydifferent orders of magnitude, and thattherefore the Special Theory is of no value intrying to explain the effect. COMPARISON OF THE SAGNAC EFFECT WITH STRSTR stipulates that the time t' recorded by an observer moving at velocity v is slower than the time to recorded by a stationary observer, according to:to = t'γwhere γ = (1 - v2/c2)-1/2 = 1 + v2/2c2 + O(v/c)4...to = t'(1 + v2/2c2)dtR = (to - t')/to = v2/(v2 + 2c2)dtR = relativity time ratioNow, to - t' = 2πr/c - 2πr/(c + v) = 2πrv/(c + v)cdt' = to - t' = tov/(c + v)dtS = (to - t')/to = v/(v + c)dtS = Sagnac ratiodtS/dtR = (2c2 + v2)/v(v + c)When v is small as compared to c, as is the case in all practical experiments, this ratioreduces to 2c/v.Thus the Sagnac effect is far larger than anypurely Relativistic effect. For example,considering the data in the Pogany test (8 ),where the rim of the disc was moving with avelocity of 25 m/s, the ratio dtS/dtR is about1.5 x 10^7. Any attempt to explain the Sagnacas a Relativistic effect is thus useless, as it issmaller by a factor of 10^7. Referring back to equation (I), consider a discof radius one kilometre. In this case a fringeshift of one fringe is achieved with a velocityat the perimeter of the disc of 0.013m/s. Thisis an extremely low velocity, being less thanlm per minute. In this case the Sagnac effectwould be 50 billion times larger than thecalculated effect under the Relativity Theory.Post (1967) shows that the two (Sagnac and STR) are of very different orders of magnitude. He says that the dilation factor to be applied under SR is “indistinguishable with presently available equipment” and “is still one order smaller than the Doppler correction, which occurs when observing fringe shifts” in the Sagnac tests. He also points out that the Doppler effect “is v/c times smaller than the effect one wants to observe." Here Post states that the effect forecast by SR, for the time dilation aboard a moving object, is far smaller than the effect to be observed in a Sagnac test. By the way, Post proved in 1999 the equivalence between the Michelson-Morley experiment and the Sagnac experiment.E. J. Post, A joint description of the Michelson Morley and Sagnac experiments.Proceedings of the International Conference Galileo Back in Italy II, Bologna 1999,Andromeda, Bologna 2000, p. 62E. J. Post is the only person to notice the substantial identity between the 1925 experiment and that of 1887: "To avoid possible confusion, it may be remarked that the beam path in the more well-known Michelson-Morley interferometer, which was mounted on a turntable, does not enclose a finite surface area; therefore no fringe shift can be expected as a result of a uniform rotation of the latter".E. J. Post, Reviews of Modern Physics. Vol. 39, n. 2, April 1967 A. Michelson and E. Morley simply measured the Coriolis effect. The Coriolis effect can be registered/recorded either due to the rotation of the Earth or due to the rotation of the ether drift (Whittaker's potential scalar waves). The deciding factor is of course the Sagnac effect, which is much greater than the Coriolis effect, and was never registered. Since MM did not use a phase-conjugate mirror or a fiber optic equipment, the Coriolis force effects upon the light offset each other. The positive (slight deviations) from the null result are due to a residual surface enclosed by the multiple path beam (the Coriolis effect registered by a Sagnac interferometer). Dayton Miller also measured the Coriolis effect of the ether drift in his experiment (Mount Wilson, 1921-1924 and 1925-1926, and Cleveland, 1922-1924). Dr. Patrick Cornille (Essays on the Formal Aspects of Electromagnetic Theory, pg. 141): https://www.osapublishing.org/ol/abstract.cfm?uri=ol-6-8-401 Sagnac effect in fiber gyroscopes H.J. Arditty and H.C. Lefevre Optics Letters, vol. 6, 1981 We review the kinematic explanation of the Sagnac effect in fiber gyroscopes and recall that the index of the dielectric medium does not have any influence. Your explanation/"theory" is based on the conventional approach to optics (the Heaviside-Lorentz equations). You need to upgrade your understanding, and use the original Maxwell equations, written in quaternion form, and which are invariant under galilean transformations. As for the Fizeau experiment (and the Fresnel drag factor) you need to study Ockert's analysis (1968 and 1969).
  4. Fig 11 refers to the Selecting Proper PCMs, NOT to Section 3, entitled Phase Conjugate Sagnac Experiment. In section 3, we have this caption: This experiment shows us two important points. First, it confirms the phase reversal of a PCM and demonstrates the Sagnac effect in an arc segment AB, not a closed path. Second, it gives us important implications: The result, φ = 4πRΩL/cλ, can be re-written as φ = 4πvL/cλ where v is the speed of the moving arc segment AB (where R is the radius of the circular motion, Ω is the rotational rate).If we increase the radius of the circular motion as shown in Fig. 6, the arc segment AB will approach a linear segment AB, the circular motion will approach the linear motion, the phase-conjugate Sagnac experiment will approach the phase-conjugate first-order experiment as shown in Fig. 4, and the phase shift is always φ = 4πvL/cλ. Take a look at the derivation, ONLY A CONTINUOUS PATH OF LIGHT. No area is present in the final formula. You have a single line/segment. The words used by the authors: "No enclosed area appears in this expression." The Sagnac effect is distributed along a line, not an enclosed area.
  5. Try again. Figure 11 refers to the section called Selecting proper PCMs. Figures 5 and 6 refer to the phase conjugate single segment of light Sagnac experiment. You have an OPEN LOOP. You cannot get to point B from point A unless the beam of light travels in the OPEN LOOP. Very simple. There is no enclosed area. The beam of light travels ONLY THROUGH THE SINGLE SEGMENT AB. Please read. This experiment shows us two important points. First, it confirms the phase reversal of a PCM and demonstrates the Sagnac effect in an arc segment AB, not a closed path. Second, it gives us important implications: The result, φ = 4πRΩL/cλ, can be re-written as φ = 4πvL/cλ where v is the speed of the moving arc segment AB (where R is the radius of the circular motion, Ω is the rotational rate).If we increase the radius of the circular motion as shown in Fig. 6, the arc segment AB will approach a linear segment AB, the circular motion will approach the linear motion, the phase-conjugate Sagnac experiment will approach the phase-conjugate first-order experiment as shown in Fig. 4, and the phase shift is always φ = 4πvL/cλ. NOT A CLOSED PATH! NO AREA. Yet, you drew, again, an area where none exists, the figure referenced is figure 6, not figure 11: the beam of light travels ONLY within segment AB. Another reference: SAGNAC effect without an area. https://arxiv.org/pdf/gr-qc/0401005.pdf
  6. I won't accuse you of trolling this thread. But I have made it clear that this subject (which was referenced while mentioning the potential) requires a different thread. These are open loops. Just like the interferometer created by Professor Yeh. A closed loop? RLGs and the MGX. The phase conjugate mirror has revolutionized the field of optics. Please study this subject a little bit more, starting here: Here is a paper written by Professor Ruyong Wang, in which the Sagnac effect is being derived without an area (what Dr. Wang calls "closed path"): https://arxiv.org/ftp/physics/papers/0609/0609202.pdf But it does not cross itself. To get from point A to point B requires an OPEN LOOP. It does matter, if there is no area enclosed. It has already been done. Here is the derivation for the SAGNAC without an area: https://arxiv.org/ftp/physics/papers/0609/0609202.pdf Here is the formula: The Sagnac effect for a ROTATING LINEAR SEGMENT interferometer IS: 2vL/c^2, where v=RΩ. No area involved at all. Just like the interferometer featured in Professor Yeh's papers.
  7. Surely you can differentiate between an OPEN LOOP and a CLOSED LOOP. The reason I brought up Professor Yeh's first paper is because in that article the OPEN LOOP is displayed right in front of reader. In fact... Starting point A An OPEN LOOP Then we arrive at point B Very clearly and precisely defined. You drew red circles over the two OPEN LOOPS. Now, if you want them to be closed at any cost, you can draw even more circles over them, but the facts won't change. There is no area whatsoever in Professor Yeh's interferometer. 2(v1l1 + v2l2)/c2Since v1 = R1 x Ω, and v2 = R2 x Ω, the formula becomes: 2(R1L1 + R2L2)Ω/c2 Since Δφ = 2πc/λ x Δt, we obtain: 4π(R1L1 + R2L2)Ω/λc which is of course Professor Yeh's formula.
  8. For the Coriolis effect derivation, one needs an area. This was proven in 1921 by Dr. Ludwik Silberstein: http://www.conspiracyoflight.com/Michelson-Gale/Silberstein.pdfThe propagation of light in rotating systems, Journal of the Optical Society of America, vol. V, number 4, 1921 For the Sagnac effect derivation, without an area, one needs to employ the use of a phase conjugate mirror. The formulas have already been derived: you have already read Professor Yeh's papers, here is another paper written by Professor Ruyong Wang, in which the Sagnac effect is being derived without an area (what Dr. Wang calls "closed path"): https://arxiv.org/ftp/physics/papers/0609/0609202.pdf You do have an open loop and closed path. Let me clarify matters even further by using Professor Yeh's first paper on the subject, where the Sagnac effect was recorded using only one arm (segment of light/open loop), as opposed to the second experiment, where two arms (open loops with different lengths/different radii) were employed. https://apps.dtic.mil/dtic/tr/fulltext/u2/a170203.pdf (page 71 of the pdf document, appendix 5.4, Phase-Conjugate Fiber Optic Gyro) To get from point A to point B and from point B to point A you need a continuous path and an open loop. This is the first experiment carried out by Professor Yeh, using only one open loop/arm. Then, we have the second experiment which uses TWO OPEN LOOPS, instead of just one. https://apps.dtic.mil/dtic/tr/fulltext/u2/a170203.pdf (page 31 of the pdf document), appendix 5.1) Two continuous paths: BS to PCM, PCM to M using two OPEN LOOPS, just like in the first experiment. No area involved at all. One does not need to use an interferometer with an area, while utilizing the phase conjugate mirror.
  9. No. The Sagnac effect is a non-relativistic effect. COMPARISON OF THE SAGNAC EFFECT WITH SPECIAL RELATIVITY, starts on page 7, calculations/formulas on page 8http://www.naturalphilosophy.org/pdf/ebooks/Kelly-TimeandtheSpeedofLight.pdfpage 8Because many investigators claim that theSagnac effect is made explicable by using theTheory of Special Relativity, a comparison ofthat theory with the actual test results is givenbelow. It will be shown that the effectscalculated under these two theories are of verydifferent orders of magnitude, and thattherefore the Special Theory is of no value intrying to explain the effect. COMPARISON OF THE SAGNAC EFFECT WITH STRSTR stipulates that the time t' recorded by an observer moving at velocity v is slower than the time to recorded by a stationary observer, according to:to = t'γwhere γ = (1 - v2/c2)-1/2 = 1 + v2/2c2 + O(v/c)4...to = t'(1 + v2/2c2)dtR = (to - t')/to = v2/(v2 + 2c2)dtR = relativity time ratioNow, to - t' = 2πr/c - 2πr/(c + v) = 2πrv/(c + v)cdt' = to - t' = tov/(c + v)dtS = (to - t')/to = v/(v + c)dtS = Sagnac ratiodtS/dtR = (2c2 + v2)/v(v + c)When v is small as compared to c, as is the case in all practical experiments, this ratioreduces to 2c/v.Thus the Sagnac effect is far larger than anypurely Relativistic effect. For example,considering the data in the Pogany test (8 ),where the rim of the disc was moving with avelocity of 25 m/s, the ratio dtS/dtR is about1.5 x 10^7. Any attempt to explain the Sagnacas a Relativistic effect is thus useless, as it issmaller by a factor of 10^7. Referring back to equation (I), consider a discof radius one kilometre. In this case a fringeshift of one fringe is achieved with a velocityat the perimeter of the disc of 0.013m/s. Thisis an extremely low velocity, being less thanlm per minute. In this case the Sagnac effectwould be 50 billion times larger than thecalculated effect under the Relativity Theory.Post (1967) shows that the two (Sagnac and STR) are of very different orders of magnitude. He says that the dilation factor to be applied under SR is “indistinguishable with presently available equipment” and “is still one order smaller than the Doppler correction, which occurs when observing fringe shifts” in the Sagnac tests. He also points out that the Doppler effect “is v/c times smaller than the effect one wants to observe." Here Post states that the effect forecast by SR, for the time dilation aboard a moving object, is far smaller than the effect to be observed in a Sagnac test.
  10. I am going to derive both the SAGNAC EFFECT and the CORIOLIS EFFECT formulas for the MGX interferometer, so that everyone will be able to see at a glance the difference. Point A is located at the detectorPoint B is in the bottom right cornerPoint C is in the upper right cornerPoint D is in the upper left cornerl1 is the upper arm.l2 is the lower arm.Here is the most important part of the derivation of the full/global Sagnac effect for an interferometer located away from the center of rotation.A > B > C > D > A is a continuous counterclockwise path, a negative sign -A > D > C > B > A is a continuous clockwise path, a positive sign +The Sagnac phase difference for the clockwise path has a positive sign.The Sagnac phase difference for the counterclockwise has a negative sign.Sagnac phase components for the A > D > C > B > A path (clockwise path):l1/(c - v1)-l2/(c + v2)Sagnac phase components for the A > B > C > D > A path (counterclockwise path):l2/(c - v2)-l1/(c + v1)For the single continuous clockwise path we add the components:l1/(c - v1) - l2/(c + v2)For the single continuous counterclockwise path we add the components:l2/(c - v2) - l1/(c + v1)The net phase difference will be (let us remember that the counterclockwise phase difference has a negative sign attached to it, that is why the substraction of the phase differences becomes an addition):{l1/(c - v1) - l2/(c + v2)} - (-){l2/(c - v2) - l1/(c + v1)} = {l1/(c - v1) - l2/(c + v2)} + {l2/(c - v2) - l1/(c + v1)}Rearranging terms:l1/(c - v1) - l1/(c + v1) + {l2/(c - v2) - l2/(c + v2)} =2(v1l1 + v2l2)/c2 This is how the correct Sagnac formula is derived: we have single continuous clockwise path, and a single continuous counterclockwise path.If we desire the Coriolis effect, we simply substract as follows:dt = l1/(c - v1) - l1/(c + v1) - (l2/(c - v2) - l2/(c + v2))Of course, by proceeding as in the usual manner for a Sagnac phase shift formula for an interferometer whose center of rotation coincides with its geometrical center, we obtain:2v1l1/(c2 - v21) - 2v2l2/(c2 - v22)l = l1 = l22l[(v1 - v2)]/c22lΩ[(R1 - R2)]/c2R1 - R2 = h2lhΩ/c2By having substracted two different Sagnac phase shifts, valid for the two different segments, we obtain the CORIOLIS EFFECT formula.However, for the SAGNAC EFFECT, we have a single CONTINUOUS CLOCKWISE PATH, and a single CONTINUOUS COUNTERCLOCKWISE PATH, as the definition of the Sagnac effect entails. This is exactly what Professor Yeh's ingenious experiment entailed: the use of the phase conjugate mirror permitted, for the first time, to actually separate the clockwise/counterclockwise paths, while at the same time the open loops have different radii and thus different linear velocities. Again, let us compare the derivations for both cases: SAGNAC and CORIOLIS. CORIOLIS EFFECT derivation dt=l1/(c - v1)+l2/(c + v2)-l1/(c + v1)-l2/(c - v2)=l1/(c - v1)-l1/(c + v1)+l2/(c + v2)-l2/(c - v2)=l1(c + v1-c + v1)/(c2 - v12)+l2(c - v2-c - v2)/(c2 - v22)=2*l1v1/(c2 - v12)-2*l2v2/(c2 - v22) dt=2*l1v1/c2-2*l2v2/c2=(2*l1v1-2*l2v2)/c2=2*(l1v1-l2v2)/c2 We are comparing two OPEN SEGMENTS: defying the very definition of the Sagnac effect.Path 1 - A>B, D>C. Path 2 - C>D, B>A By comparison, the Sagnac phase components for the A > D > C > B > A path (clockwise path):l1/(c - v1)-l2/(c + v2)Sagnac phase components for the A > B > C > D > A path (counterclockwise path):l2/(c - v2)-l1/(c + v1)For the single continuous clockwise path we add the components:l1/(c - v1) - l2/(c + v2)For the single continuous counterclockwise path we add the components:l2/(c - v2) - l1/(c + v1) 2(v1l1 + v2l2)/c2Since v1 = R1 x Ω, and v2 = R2 x Ω, the formula becomes: 2(R1L1 + R2L2)Ω/c2 Since Δφ = 2πc/λ x Δt, we obtain: 4π(R1L1 + R2L2)Ω/λc which is of course Professor Yeh's formula. FULL CORIOLIS EFFECT FOR THE MGX:4AΩsinΦ/c2FULL SAGNAC EFFECT FOR THE MGX:4Lv(cos2Φ1 + cos2Φ2)/c2Sagnac effect/Coriolis effect ratio:R((cos2Φ1 + cos2Φ2)/hsinΦ R = 4,250 kmh = 0.33924 kmThe rotational Sagnac effect is much greater than the Coriolis effect for the MGX.Φ1 = Φ = 41°46' = 41.76667°Φ2 = 41°45' = 41.75°R((cos2Φ1 + cos2Φ2) = 4729.885hsinΦ = 0.2259674729.885/0.225967 = 20,931.72 A Sagnac light interferometer (MGX, RLGs) can detect BOTH EFFECTS: one is a physical effect proportional to the area - the CORIOLIS EFFECT; the other one is an electromagnetic effect proportional to the radius of rotation - the SAGNAC EFFECT.
  11. Those are open loops, and you know this fact very well. Not closed loops. No area. No enclosure whatsoever. (Open-ended (non-closed) loops: a single segment from end to end) Loop = a structure, series, or process, the end of which is connected to the beginning. The use of the phase conjugate mirror means that you have an interferometer where there is no need to actually have an enclosed area. The paths of light in the figure are the two segments of light which connect BS and M with PCM, back and forth. The path of the light is described as follows: Light from a laser is split into two separate fibers, F1 and F2 which are coiled such that light travels clockwise in F1 and counterclockwise in F2. Both of you already know these very simple facts. You have TWO CLOSED PATHS and TWO OPEN LOOPS. Did you even read the paper? The description is very clear: https://apps.dtic.mil/dtic/tr/fulltext/u2/a170203.pdf (page 34 of the pdf document) where R1,2 and L1,2 are the radii and lengths of the fiber loops, and Ω is the rotation rate. You have two OPEN LOOPS of radii R1,2. The lengths of the paths of light are L1,2, and they are different for each open loop. Very simple. But it is. φ = -2(φ2 - φ1) = 4π(R1L1 + R2L2)Ω/λc V1 = R1 x Ω V2 = R2 x Ω Do you understand this very easy substitution? 4π(V1L1 + V2L2)/λcSince Δφ = 2πc/λ x Δt, Δt = 2(R1L1 + R2L2)Ω/c2 = 2(V1L1 + V2L2)/c2CORRECT SAGNAC FORMULA:2(V1L1 + V2L2)/c2 But it is the same. You have two different radii, thus two different linear velocities. Exactly the same case as in the Michelson-Gale experiment, or the ring laser gyroscopes interferometers. This is why the experiment carried out by Dr. Yeh is so ingenious. It is the same. φ = -2(φ2 - φ1) = 4π(R1L1 + R2L2)Ω/λc V1 = R1 x Ω V2 = R2 x Ω 4π(V1L1 + V2L2)/λc You have two different velocities, for each latitude, in the Michelson-Gale experiment. Each open loop has a different linear velocity, since both open loops have different radii, in the figure shown by Professor Yeh. Remember, we are dealing with a Sagnac interferometer which is located away from the center of rotation (as in the case of all ring laser gyroscopes or the MGX). Then, you will automatically have two different linear velocities, for each arm, and two different lengths of the arms. The Sagnac effect is an electromagnetic effect. https://www.researchgate.net/publication/253792325_Sagnac_Effect What S.A. Werner measured in 1979 is the CORIOLIS EFFECT upon the neutron phase: https://arxiv.org/pdf/1701.00259.pdf Once the area of the interferometer is mentioned you get the CORIOLIS EFFECT. If you want the SAGNAC EFFECT, you must derive a formula which is proportional to the RADIUS OF ROTATION, just like I did, just like Professor Yeh did. The derivation I provided obeys the RULES of the Sagnac effect, while Michelson's derivation did not. He compared two open arms of the interferometer, thus obtaining the Coriolis effect formula. I compared two loops, one counterclockwise, one clockwise, thus I obtained the correct Sagnac effect formula.
  12. No speculation at all. Just a straightforward and direct derivation, using the correct definition of the Sagnac effect. Remember this: the CORIOLIS EFFECT is a physical effect upon the light beams. It is directly proportional to the area of the interferometer. The SAGNAC EFFECT is an electromagnetic effect upon the velocities of the light beams, as such it must be directly proportional to the RADIUS of rotation. A huge difference. As for the subquarks, I should open a new thread, where we will investigate the nature of the potential, which, as R. Feynman stated, is much more important than the vector field.
  13. Don't you want to know what the potential consists of? What exactly causes the Aharonov-Bohm effect? How do subquarks relate to this very important branch of physics? Perhaps I will open a new thread, in the Speculations section of course, devoted to this subject. For now, please study the topological implications of the Aharonov-Bohm effect: http://redshift.vif.com/JournalFiles/Pre2001/V07NO1PDF/V07N1BAR.pdf (Dr. Terence W. Barrett, Stanford University) None of my assertions have been contradicted. Do you understand how a phase conjugate mirror functions? There is no area at all featured in Professor Yeh's interferometer, just two segments of light. Please show to your readers where the area is in the following diagram: But it is. Surely you know that v = R x Ω. This is the formula published by Professor Yeh: φ = -2(φ2 - φ1) = 4π(R1L1 + R2L2)Ω/λc V1 = R1 x Ω V2 = R2 x Ω Do you understand this very easy substitution? 4π(V1L1 + V2L2)/λcSince Δφ = 2πc/λ x Δt, Δt = 2(R1L1 + R2L2)Ω/c2 = 2(V1L1 + V2L2)/c2CORRECT SAGNAC FORMULA:2(V1L1 + V2L2)/c2The very same formula obtained for a Sagnac interferometer which features two different lengths and two different velocities.
  14. Here is more "voodoo" for you: the Maxwell-Lodge effect. http://citeseerx.ist.psu.edu/viewdoc/download?doi=10.1.1.726.6101&rep=rep1&type=pdfThe Maxwell-Lodge effect: significance of electromagnetic potentials in the classical theoryG. Rousseaux, R. Kofman, and O. MinazzoliThe Aharonov-Bohm effect has been the starting point of the reconsideration of the reality of the vector potential within quantum physics. We argue that the Maxwell-Lodge effect is its classical equivalent: what is the origin of the electromotive force induced in a coil surrounding a (finite) solenoid fed by an alternative current? We demonstrate theoretically, experimentally and numerically that the effect can be understood using the vector potential while it cannot using only the fields. http://www.ccsenet.org/journal/index.php/apr/article/view/26623/17220The Physical Entity of Vector Potential in ElectromagnetismVladimir A. Leus, Ray T. Smith and Simon Maher “...the vector potential appears to give the most direct description of the physics. This becomes more apparent the more deeply we go into quantum theory. In the general theory of quantum electrodynamics, one takes the vector and scalar potentials as the fundamental quantities in a set of equations that replace the Maxwell equations: E and B are slowly disappearing from the modern expression of physical laws; they are being replaced by A and φ” (Feynman et al, 1989, chapter 15, section 5, The Feynman Lecture on Physics (Vol. 2), 1989) Did you know that there is also a Gravitational Aharonov-Bohm effect? But you haven't done your homework here. You treated the entire subject matter superficially; hopefully, the attention you devote to your other areas of interest will be more focused.
  15. I made no such suggestion. I did not even mention photons. I mentioned scalar waves, not scalar fields (aether). Are you going to call the Aharonov-Bohm voodoo physics? It is being caused by the POTENTIAL, in the absence of vector fields. Are you going to call Whittaker's proofs as voodoo physics? He proved, mathematically, the existence of scalar/longitudinal waves. Are you going to call Maxwell's original set of equations, which are invariant under galilean transformations voodoo physics? You better not. Now, for those interested in the correct model of the atom, which does include the subquark, I can open a new thread, again with definite proofs. You will find out about Martin Ruderfer's celebrated experiment (1960), the first null result in the history of ether drift analysis, how the Riemann zeta function is related to the mass of a boson, and much more. But this would be the subject of a different thread. Here we are concerned with the Sagnac effect formula for an interferometer which features different velocities for each arm, and different lengths for each arm. Go ahead and make my day: prove that my derivation is wrong. If you cannot, then you must replace the word speculations with proofs.
×
×
  • Create New...

Important Information

We have placed cookies on your device to help make this website better. You can adjust your cookie settings, otherwise we'll assume you're okay to continue.